LSAT and Law School Admissions Forum

Get expert LSAT preparation and law school admissions advice from PowerScore Test Preparation.

User avatar
 Dave Killoran
PowerScore Staff
  • PowerScore Staff
  • Posts: 5972
  • Joined: Mar 25, 2011
|
#88172
Complete Question Explanation
(The complete setup for this game can be found here: lsat/viewtopic.php?f=170&p=88166#p88166)

The correct answer choice is (A)

The question stem in this problem establishes that H receives two stars, and that H is the only CD to receive two stars. From the second rule we can determine that N receives just one star:

G2-Q7-d1.png

This information eliminates answer choice (A) from contention.

From the third rule, we can determine that I must receive the same rating as R. Pairing that inference with the deductions made in the discussion of the fourth rule, we can determine that I and R receive a rating of three stars, and that Q receives a rating of four stars:

G2-Q7-d2.png

Not surprisingly, S, the only random in the game, is not placed, although it must receive a rating of either one star or four stars.

As we have determined that I must receive a rating of three stars, answer choice (A) is proven correct.

Answer choices (B), (C), and (D) can never occur. Answer choice (E) is possible but does not have to occur.
You do not have the required permissions to view the files attached to this post.
 Adam Tyson
PowerScore Staff
  • PowerScore Staff
  • Posts: 5389
  • Joined: Apr 14, 2011
|
#74756
The correct answer choice is (A)

In this local question, H must be the only variable in the 2nd slot. Because of the NH block, this forces N into the first slot. Also, the third rule, requiring that I be paired with either H or R, now forces I to be with R, and they can only go in the third slot (if that pair is placed in the 4th slot we will violate the last rule about no more than one CD getting more stars than Q gets). This ultimately forces Q into the fourth slot. S can now be in either slot 1, with N, or else in slot 4, with Q (that last rule, about Q, DOES allow Q to be tied with another variable).

Answer choice (A): This is the correct answer.

Answer choice (B): This answer choice cannot be true.

Answer choice (C): This answer choice cannot be true.

Answer choice (D): This answer choice cannot be true.

Answer choice (E): This answer choice could be true but does not have to be true.
 ncolicci11
  • Posts: 43
  • Joined: Feb 09, 2020
|
#74749
Powerscore,

Could you review this question? I understand why A is right, but wouldn't this scenario force Sounds good to be in slot 1 with N?
 ncolicci11
  • Posts: 43
  • Joined: Feb 09, 2020
|
#74750
Actually... Sounds good could also be in slot 4 with Q, right? This is because it at most has another CD one star higher so it can be in four with another CD.
 Adam Tyson
PowerScore Staff
  • PowerScore Staff
  • Posts: 5389
  • Joined: Apr 14, 2011
|
#74755
Exactly right! S could get 1 star (the same as N) or else 4 stars (the same as Q). The rule about Q does allow some other CD to get as many stars as it does. Well done! That's why answer E does not have to be true, even though it could be true.

Get the most out of your LSAT Prep Plus subscription.

Analyze and track your performance with our Testing and Analytics Package.